Show that $mathbb{Q}(sqrt{5}+sqrt[3]{2})=mathbb{Q}(sqrt{5},sqrt[3]{2})$.












8












$begingroup$



Show that $mathbb{Q}(sqrt{5}+sqrt[3]{2})=mathbb{Q}(sqrt{5},sqrt[3]{2})$.




I've got that $big[mathbb{Q}(sqrt{5}+sqrt[3]{2}):mathbb{Q}big] in {1,2,3,6}$ because it's going to divide $big[mathbb{Q}(sqrt{5},sqrt[3]{2}):mathbb{Q}big]=6$. Clearly it is not $1$. I want to show that it is not $2$ or $3$. So I'm saying that if it is $2$ then $alpha = sqrt{5}+sqrt[3]{2}$ satisfies the relation
$$alpha^2 + balpha = k$$ for $b,k in mathbb{Q}$ since $alpha$ will be the root of a monic irreducible polynomial of degree $2$. How can I obtain a contradiction from this? I also need to do the degree $3$ case somehow.



Also if there is a better way to do this than what I'm doing I'd be excited to learn about it.










share|cite|improve this question











$endgroup$








  • 1




    $begingroup$
    There is a fairly general result in this direction, for fields of characteristic $0$. See the accepted answer at mathoverflow.net/questions/26832/….
    $endgroup$
    – KCd
    Jun 24 '15 at 4:07
















8












$begingroup$



Show that $mathbb{Q}(sqrt{5}+sqrt[3]{2})=mathbb{Q}(sqrt{5},sqrt[3]{2})$.




I've got that $big[mathbb{Q}(sqrt{5}+sqrt[3]{2}):mathbb{Q}big] in {1,2,3,6}$ because it's going to divide $big[mathbb{Q}(sqrt{5},sqrt[3]{2}):mathbb{Q}big]=6$. Clearly it is not $1$. I want to show that it is not $2$ or $3$. So I'm saying that if it is $2$ then $alpha = sqrt{5}+sqrt[3]{2}$ satisfies the relation
$$alpha^2 + balpha = k$$ for $b,k in mathbb{Q}$ since $alpha$ will be the root of a monic irreducible polynomial of degree $2$. How can I obtain a contradiction from this? I also need to do the degree $3$ case somehow.



Also if there is a better way to do this than what I'm doing I'd be excited to learn about it.










share|cite|improve this question











$endgroup$








  • 1




    $begingroup$
    There is a fairly general result in this direction, for fields of characteristic $0$. See the accepted answer at mathoverflow.net/questions/26832/….
    $endgroup$
    – KCd
    Jun 24 '15 at 4:07














8












8








8


4



$begingroup$



Show that $mathbb{Q}(sqrt{5}+sqrt[3]{2})=mathbb{Q}(sqrt{5},sqrt[3]{2})$.




I've got that $big[mathbb{Q}(sqrt{5}+sqrt[3]{2}):mathbb{Q}big] in {1,2,3,6}$ because it's going to divide $big[mathbb{Q}(sqrt{5},sqrt[3]{2}):mathbb{Q}big]=6$. Clearly it is not $1$. I want to show that it is not $2$ or $3$. So I'm saying that if it is $2$ then $alpha = sqrt{5}+sqrt[3]{2}$ satisfies the relation
$$alpha^2 + balpha = k$$ for $b,k in mathbb{Q}$ since $alpha$ will be the root of a monic irreducible polynomial of degree $2$. How can I obtain a contradiction from this? I also need to do the degree $3$ case somehow.



Also if there is a better way to do this than what I'm doing I'd be excited to learn about it.










share|cite|improve this question











$endgroup$





Show that $mathbb{Q}(sqrt{5}+sqrt[3]{2})=mathbb{Q}(sqrt{5},sqrt[3]{2})$.




I've got that $big[mathbb{Q}(sqrt{5}+sqrt[3]{2}):mathbb{Q}big] in {1,2,3,6}$ because it's going to divide $big[mathbb{Q}(sqrt{5},sqrt[3]{2}):mathbb{Q}big]=6$. Clearly it is not $1$. I want to show that it is not $2$ or $3$. So I'm saying that if it is $2$ then $alpha = sqrt{5}+sqrt[3]{2}$ satisfies the relation
$$alpha^2 + balpha = k$$ for $b,k in mathbb{Q}$ since $alpha$ will be the root of a monic irreducible polynomial of degree $2$. How can I obtain a contradiction from this? I also need to do the degree $3$ case somehow.



Also if there is a better way to do this than what I'm doing I'd be excited to learn about it.







abstract-algebra polynomials field-theory extension-field irreducible-polynomials






share|cite|improve this question















share|cite|improve this question













share|cite|improve this question




share|cite|improve this question








edited Dec 18 '18 at 0:40









Batominovski

33.1k33293




33.1k33293










asked Jun 24 '15 at 1:53









TuoTuoTuoTuo

1,776516




1,776516








  • 1




    $begingroup$
    There is a fairly general result in this direction, for fields of characteristic $0$. See the accepted answer at mathoverflow.net/questions/26832/….
    $endgroup$
    – KCd
    Jun 24 '15 at 4:07














  • 1




    $begingroup$
    There is a fairly general result in this direction, for fields of characteristic $0$. See the accepted answer at mathoverflow.net/questions/26832/….
    $endgroup$
    – KCd
    Jun 24 '15 at 4:07








1




1




$begingroup$
There is a fairly general result in this direction, for fields of characteristic $0$. See the accepted answer at mathoverflow.net/questions/26832/….
$endgroup$
– KCd
Jun 24 '15 at 4:07




$begingroup$
There is a fairly general result in this direction, for fields of characteristic $0$. See the accepted answer at mathoverflow.net/questions/26832/….
$endgroup$
– KCd
Jun 24 '15 at 4:07










4 Answers
4






active

oldest

votes


















3












$begingroup$

Since you know that $[mathbb Q[sqrt[3]{2},sqrt 5]:mathbb Q] = 6$, you know that each of the six values $$1,sqrt{5},\sqrt[3]{2},sqrt[3]{2}sqrt{5},\sqrt[3]{4},sqrt[3]{4}sqrt 5tag{1}$$
are linearly independent over $mathbb Q$.



Now $$(sqrt[3]2+sqrt 5)^2=5cdot 1 + 2cdot sqrt[3]2sqrt 5 + 1cdotsqrt[3]4$$



Is it possible for $1,sqrt[3]2+sqrt5,(sqrt[3]2+sqrt 5)^2$ to be linearly dependent over $mathbb Q$?



Do the same with by adding the cube $(sqrt[3]2+sqrt5)^3$.



Another way to look at it use (1) as a basis, and write elements of the field as:



$$(a,b,c,d,e,f)to acdot 1 + bcdot sqrt5+csqrt[3]2+dsqrt[3]2sqrt5+esqrt[3]4+fsqrt[3]4sqrt5$$



Then $$begin{align}(1,0,0,0,0,0)&leftrightarrow 1\(0,1,1,0,0,0)&leftrightarrow sqrt 5+ sqrt[3]2\(5,0,0,2,1,0)&leftrightarrow (sqrt5+sqrt[3]2)^2\
(2,5,15,0,0,3)&leftrightarrow (sqrt5+sqrt[3]2)^3
end{align}$$



And those four vectors are "obviously" linearly independent.






share|cite|improve this answer











$endgroup$









  • 1




    $begingroup$
    You don't need to go all the way up to $j=5$, since you know that it is either $1,2,3,$ or $6$. No reason to go beyond the cube. @orangeskid That conclusion was in the question. You only need to show that it is not a root of a quadratic, cubic or linear polynomial. That makes it very easy to prove linear independency of the above.
    $endgroup$
    – Thomas Andrews
    Jun 24 '15 at 2:33












  • $begingroup$
    Ah, OK, I get it. That's clever.
    $endgroup$
    – Orest Bucicovschi
    Jun 24 '15 at 2:43












  • $begingroup$
    I like this. I think I almost had it but didn't think to use the linear independence of the basis over $mathbb{Q}$.
    $endgroup$
    – TuoTuo
    Jun 24 '15 at 17:08





















3












$begingroup$

Let's show that $s=sum sqrt[n_i]{d_i}$ ($d_i >0$ rationals ) generates $mathbb{Q}(sqrt[n_i]{d_i})$. Consider all in larger Galois extension $K supset mathbb{Q}(sqrt[n_i]{d_i})supset mathbb{Q}$. Now, to show that $s$ generates all the $sqrt[n_i]{d_i}$ it's enough to show that whenever a Galois transformation $phi$ of $K$ preserves $s$ it must preserve all the $sqrt[n_i]{d_i}$. Now any Galois transformation $phi$ takes $sqrt[n_i]{d_i}$ to some $omega_i sqrt[n_i]{d_i}$ where $omega_i^{n_i} =1$, so $phi(s) = sum omega_i sqrt[n_i]{d_i}$.
Note that
$$left|sum_i omega_i sqrt[n_i]{d_i}right| le sum_i sqrt[n_i]{d_i}$$ with equality
if and only if $omega_i$ have the same argument. Therefore, if $sum_i omega_i sqrt[n_i]{d_i}= sum_i sqrt[n_i]{d_i}$ then all the $omega_i$ must be $1$.



Conclusion: $phi(s) = s implies phi( sqrt[n_i]{d_i}) = sqrt[n_i]{d_i}$ for all $i$.



$bf{Added:}$ A solution that uses mostly linear algebra, inspired from @
Thomas Andrews: 's solution.



We have the equalities
begin{eqnarray}
left(begin{array} {c}
1\sqrt[3]{2}+sqrt{5}\ (sqrt[3]{2}+sqrt{5})^2\ (sqrt[3]{2}+sqrt{5})^3\ (sqrt[3]{2}+sqrt{5})^4\ (sqrt[3]{2}+sqrt{5})^5
end{array}right) = left(
begin{array}{cccccc}
1 & 0 & 0 & 0 & 0 & 0 \
0 & 1 & 0 & 1 & 0 & 0 \
5 & 0 & 1 & 0 & 2 & 0 \
2 & 15 & 0 & 5 & 0 & 3 \
25 & 2 & 30 & 8 & 20 & 0 \
100 & 125 & 2 & 25 & 10 & 50 \
end{array}
right) cdot left(begin{array} {c} 1 \ sqrt[3]{2}\ sqrt[3]{4} \ sqrt{5} \ sqrt[3]{2} sqrt{5} \sqrt[3]{4} sqrt{5}
end{array}
right)
end{eqnarray}
hence
begin{eqnarray}
left(begin{array} {c} 1 \ sqrt[3]{2}\ sqrt[3]{4} \ sqrt{5} \ sqrt[3]{2} sqrt{5} \sqrt[3]{4} sqrt{5}
end{array}
right)
= left(
begin{array}{cccccc}
1 & 0 & 0 & 0 & 0 & 0 \
frac{2275}{4054} & -frac{1714}{2027} & frac{195}{2027} & frac{500}{2027} & -frac{9}{4054} & -frac{30}{2027} \
frac{2875}{2027} & -frac{1325}{2027} & -frac{955}{2027} & frac{150}{2027} & frac{100}{2027} & -frac{9}{2027} \
-frac{2275}{4054} & frac{3741}{2027} & -frac{195}{2027} & -frac{500}{2027} & frac{9}{4054} & frac{30}{2027} \
-frac{6505}{2027} & frac{1325}{4054} & frac{1491}{2027} & -frac{75}{2027} & -frac{50}{2027} & frac{9}{4054} \
-frac{5143}{2027} & frac{2335}{2027} & -frac{650}{2027} & -frac{991}{2027} & frac{15}{2027} & frac{100}{2027} \
end{array}
right) cdot left(begin{array} {c}
1\sqrt[3]{2}+sqrt{5}\ (sqrt[3]{2}+sqrt{5})^2\ (sqrt[3]{2}+sqrt{5})^3\ (sqrt[3]{2}+sqrt{5})^4\ (sqrt[3]{2}+sqrt{5})^5
end{array}right)
end{eqnarray}



For instance, we have



$
tiny
sqrt{5} = -frac{2275}{4054} + frac{3741}{2027} (sqrt[3]{2}+sqrt{5}) -frac{195}{2027}(sqrt[3]{2}+sqrt{5})^2 -frac{500}{2027} (sqrt[3]{2}+sqrt{5})^3+ frac{9}{4054} (sqrt[3]{2}+sqrt{5})^4+ frac{30}{2027} (sqrt[3]{2}+sqrt{5})^5$






share|cite|improve this answer











$endgroup$





















    3












    $begingroup$

    Brute-Force Method



    Let $alpha:=sqrt{5}+sqrt[3]{2}$. Then, $alpha^3-3sqrt{5}alpha^2+15alpha-5sqrt{5}=(alpha-sqrt{5})^3=2$, whence $$left(alpha^3+15alpha-2right)^3=5left(3alpha^2+5right)^2,,$$ or $alpha$ is a root of the polynomial $$f(x):=x^6-15x^4-4x^3+75x^2-60x-121 in mathbb{Q}[x],.$$ If this polynomial is reducible, then consider it modulo $3$, so $$f(x)=x^6+2x^3+2=left(x^2+2x+2right)^3$$ in $mathbb{F}_3$, and $x^2+2x+2$ is an irreducible element of $mathbb{F}_3[x]$. That is, $f(x)$ must have a monic quadratic factor $g(x)$ in $mathbb{Q}[x]$, and by Gauss's Lemma, $g(x)inmathbb{Z}[x]$. Hence, $g(x)=x^2+ax+b$ with $a,binmathbb{Z}$ and $bequiv 2 equiv -1pmod{3}$. Let $f(x)=g(x),h(x)$ for some $h(x)inmathbb{Q}[x]$ (which again yields $h(x)inmathbb{Z}[x]$). Since the coefficient of $x^5$ in $f(x)$ is $0$, $h(x)=x^4-ax^3+cx^2+dx+e$ for some $c,d,einmathbb{Z}$.



    Now, $$f(x)=x^6-4x^3+4=left(x^3-2right)^2=left(x^3+2^3right)^2=(x+2)^2left(x^2-2x+4right)^2$$ in $mathbb{F}_5$, where both $x+2$ and $x^2-2x+4$ are irreducible elements of $mathbb{F}_5[x]$. Hence, in $mathbb{F}_5$, either $$g(x)=(x+2)^2=x^2+4x+4text{ or }g(x)=x^2-2x+4,.$$ In either case, $bequiv 4equiv -1pmod{5}$. Together with $bequiv -1pmod{3}$, we deduce that $b equiv -1pmod{15}$. However, $b$ must divide the constant term $-121=-11^2$ of $f(x)$. This means $b=-1$ or $b=-121$.



    If $b=-1$, then $g(x)=x^2+ax-1$ and $h(x)=x^4-ax^3+cx^2+dx+121$. Equating the coefficients of $f(x)$ and $g(x),h(x)$, we have $$c-a^2-1=-15,,,, ac+a+d=-4,,$$ $$ad-c+121=75,,text{ and }121a-d=-60,.$$ Consequently, $c=a^2-14$ and $d=121a+60$, so that $$begin{align}0&=(ad-c+121)-75=ad-c+46\&=a(121a+60)-left(a^2-14right)+46=60left(2a^2+a+1right),.end{align}$$ However, $2a^2+a+1=0$ does not have an integer solution.



    If $b=-121$, $g(x)=x^2+ax-121$ and $h(x)=x^4-ax^3+cx^2+dx+1$. Equating the coefficients of $f(x)$ and $g(x),h(x)$, we have $$c-a^2-121=-15,,,, ac+121a+d=-4,,$$ $$ad-121c+1=75,,text{ and }a-121d=-60,.$$ Consequently, $c=a^2+106$ and $d=frac{a+60}{121}$, so that $$begin{align}0&=(ad-121c+1)-75=ad-121c-74\&=frac{a}{121}(a+60)-121left(a^2+106right)-75,,end{align}$$ or $$0=-frac{1}{121}left(14640a^2-60a+1561021right),.$$ However, $14640a^2-60a+1561021=0$ does not have an integer solution.



    We have derived a contradiction from the hypothesis that $f(x)$ is reducible over $mathbb{Q}$, so $f(x)$ must be irreducible in $mathbb{Q}[x]$. Hence, $mathbb{Q}(alpha)cong mathbb{Q}[x]/big(f(x)big)$ is a field extension of index $deg(f)=6$ over $mathbb{Q}$.






    share|cite|improve this answer











    $endgroup$













    • $begingroup$
      I think this is pretty complicated but appreciate the answer.
      $endgroup$
      – TuoTuo
      Jun 24 '15 at 17:00



















    2












    $begingroup$

    Clearly $LHSsubseteq RHS$. Now it suffices to write down the minimal polynomial of $sqrt{5}+sqrt[3]{2}$ and note it has degree $6$.






    share|cite|improve this answer









    $endgroup$













    • $begingroup$
      This would be my method, too. You get a polynomial of degree $6$, but you must also show that it’s irreducible.
      $endgroup$
      – Lubin
      Jun 24 '15 at 16:40










    • $begingroup$
      Claiming that the minimal polynomial has degree 6 with no justification is almost like assuming that these extensions are equal.
      $endgroup$
      – TuoTuo
      Jun 24 '15 at 16:47










    • $begingroup$
      Isn't there a general result saying that if $a$ has deg 2 min poly and $b$ has deg $3$ min poly, then $a+b$ has deg 6 min poly?
      $endgroup$
      – Damian Reding
      Jun 24 '15 at 17:33










    • $begingroup$
      @Mathgemnini There very well may be but I've never seen it and a quick google search didn't reveal it. In either case I doubt we could get away with using that theorem on the qual (if such a theorem exists) since it basically trivializes the problem and is not a major result.
      $endgroup$
      – TuoTuo
      Jun 24 '15 at 20:52











    Your Answer





    StackExchange.ifUsing("editor", function () {
    return StackExchange.using("mathjaxEditing", function () {
    StackExchange.MarkdownEditor.creationCallbacks.add(function (editor, postfix) {
    StackExchange.mathjaxEditing.prepareWmdForMathJax(editor, postfix, [["$", "$"], ["\\(","\\)"]]);
    });
    });
    }, "mathjax-editing");

    StackExchange.ready(function() {
    var channelOptions = {
    tags: "".split(" "),
    id: "69"
    };
    initTagRenderer("".split(" "), "".split(" "), channelOptions);

    StackExchange.using("externalEditor", function() {
    // Have to fire editor after snippets, if snippets enabled
    if (StackExchange.settings.snippets.snippetsEnabled) {
    StackExchange.using("snippets", function() {
    createEditor();
    });
    }
    else {
    createEditor();
    }
    });

    function createEditor() {
    StackExchange.prepareEditor({
    heartbeatType: 'answer',
    autoActivateHeartbeat: false,
    convertImagesToLinks: true,
    noModals: true,
    showLowRepImageUploadWarning: true,
    reputationToPostImages: 10,
    bindNavPrevention: true,
    postfix: "",
    imageUploader: {
    brandingHtml: "Powered by u003ca class="icon-imgur-white" href="https://imgur.com/"u003eu003c/au003e",
    contentPolicyHtml: "User contributions licensed under u003ca href="https://creativecommons.org/licenses/by-sa/3.0/"u003ecc by-sa 3.0 with attribution requiredu003c/au003e u003ca href="https://stackoverflow.com/legal/content-policy"u003e(content policy)u003c/au003e",
    allowUrls: true
    },
    noCode: true, onDemand: true,
    discardSelector: ".discard-answer"
    ,immediatelyShowMarkdownHelp:true
    });


    }
    });














    draft saved

    draft discarded


















    StackExchange.ready(
    function () {
    StackExchange.openid.initPostLogin('.new-post-login', 'https%3a%2f%2fmath.stackexchange.com%2fquestions%2f1337071%2fshow-that-mathbbq-sqrt5-sqrt32-mathbbq-sqrt5-sqrt32%23new-answer', 'question_page');
    }
    );

    Post as a guest















    Required, but never shown

























    4 Answers
    4






    active

    oldest

    votes








    4 Answers
    4






    active

    oldest

    votes









    active

    oldest

    votes






    active

    oldest

    votes









    3












    $begingroup$

    Since you know that $[mathbb Q[sqrt[3]{2},sqrt 5]:mathbb Q] = 6$, you know that each of the six values $$1,sqrt{5},\sqrt[3]{2},sqrt[3]{2}sqrt{5},\sqrt[3]{4},sqrt[3]{4}sqrt 5tag{1}$$
    are linearly independent over $mathbb Q$.



    Now $$(sqrt[3]2+sqrt 5)^2=5cdot 1 + 2cdot sqrt[3]2sqrt 5 + 1cdotsqrt[3]4$$



    Is it possible for $1,sqrt[3]2+sqrt5,(sqrt[3]2+sqrt 5)^2$ to be linearly dependent over $mathbb Q$?



    Do the same with by adding the cube $(sqrt[3]2+sqrt5)^3$.



    Another way to look at it use (1) as a basis, and write elements of the field as:



    $$(a,b,c,d,e,f)to acdot 1 + bcdot sqrt5+csqrt[3]2+dsqrt[3]2sqrt5+esqrt[3]4+fsqrt[3]4sqrt5$$



    Then $$begin{align}(1,0,0,0,0,0)&leftrightarrow 1\(0,1,1,0,0,0)&leftrightarrow sqrt 5+ sqrt[3]2\(5,0,0,2,1,0)&leftrightarrow (sqrt5+sqrt[3]2)^2\
    (2,5,15,0,0,3)&leftrightarrow (sqrt5+sqrt[3]2)^3
    end{align}$$



    And those four vectors are "obviously" linearly independent.






    share|cite|improve this answer











    $endgroup$









    • 1




      $begingroup$
      You don't need to go all the way up to $j=5$, since you know that it is either $1,2,3,$ or $6$. No reason to go beyond the cube. @orangeskid That conclusion was in the question. You only need to show that it is not a root of a quadratic, cubic or linear polynomial. That makes it very easy to prove linear independency of the above.
      $endgroup$
      – Thomas Andrews
      Jun 24 '15 at 2:33












    • $begingroup$
      Ah, OK, I get it. That's clever.
      $endgroup$
      – Orest Bucicovschi
      Jun 24 '15 at 2:43












    • $begingroup$
      I like this. I think I almost had it but didn't think to use the linear independence of the basis over $mathbb{Q}$.
      $endgroup$
      – TuoTuo
      Jun 24 '15 at 17:08


















    3












    $begingroup$

    Since you know that $[mathbb Q[sqrt[3]{2},sqrt 5]:mathbb Q] = 6$, you know that each of the six values $$1,sqrt{5},\sqrt[3]{2},sqrt[3]{2}sqrt{5},\sqrt[3]{4},sqrt[3]{4}sqrt 5tag{1}$$
    are linearly independent over $mathbb Q$.



    Now $$(sqrt[3]2+sqrt 5)^2=5cdot 1 + 2cdot sqrt[3]2sqrt 5 + 1cdotsqrt[3]4$$



    Is it possible for $1,sqrt[3]2+sqrt5,(sqrt[3]2+sqrt 5)^2$ to be linearly dependent over $mathbb Q$?



    Do the same with by adding the cube $(sqrt[3]2+sqrt5)^3$.



    Another way to look at it use (1) as a basis, and write elements of the field as:



    $$(a,b,c,d,e,f)to acdot 1 + bcdot sqrt5+csqrt[3]2+dsqrt[3]2sqrt5+esqrt[3]4+fsqrt[3]4sqrt5$$



    Then $$begin{align}(1,0,0,0,0,0)&leftrightarrow 1\(0,1,1,0,0,0)&leftrightarrow sqrt 5+ sqrt[3]2\(5,0,0,2,1,0)&leftrightarrow (sqrt5+sqrt[3]2)^2\
    (2,5,15,0,0,3)&leftrightarrow (sqrt5+sqrt[3]2)^3
    end{align}$$



    And those four vectors are "obviously" linearly independent.






    share|cite|improve this answer











    $endgroup$









    • 1




      $begingroup$
      You don't need to go all the way up to $j=5$, since you know that it is either $1,2,3,$ or $6$. No reason to go beyond the cube. @orangeskid That conclusion was in the question. You only need to show that it is not a root of a quadratic, cubic or linear polynomial. That makes it very easy to prove linear independency of the above.
      $endgroup$
      – Thomas Andrews
      Jun 24 '15 at 2:33












    • $begingroup$
      Ah, OK, I get it. That's clever.
      $endgroup$
      – Orest Bucicovschi
      Jun 24 '15 at 2:43












    • $begingroup$
      I like this. I think I almost had it but didn't think to use the linear independence of the basis over $mathbb{Q}$.
      $endgroup$
      – TuoTuo
      Jun 24 '15 at 17:08
















    3












    3








    3





    $begingroup$

    Since you know that $[mathbb Q[sqrt[3]{2},sqrt 5]:mathbb Q] = 6$, you know that each of the six values $$1,sqrt{5},\sqrt[3]{2},sqrt[3]{2}sqrt{5},\sqrt[3]{4},sqrt[3]{4}sqrt 5tag{1}$$
    are linearly independent over $mathbb Q$.



    Now $$(sqrt[3]2+sqrt 5)^2=5cdot 1 + 2cdot sqrt[3]2sqrt 5 + 1cdotsqrt[3]4$$



    Is it possible for $1,sqrt[3]2+sqrt5,(sqrt[3]2+sqrt 5)^2$ to be linearly dependent over $mathbb Q$?



    Do the same with by adding the cube $(sqrt[3]2+sqrt5)^3$.



    Another way to look at it use (1) as a basis, and write elements of the field as:



    $$(a,b,c,d,e,f)to acdot 1 + bcdot sqrt5+csqrt[3]2+dsqrt[3]2sqrt5+esqrt[3]4+fsqrt[3]4sqrt5$$



    Then $$begin{align}(1,0,0,0,0,0)&leftrightarrow 1\(0,1,1,0,0,0)&leftrightarrow sqrt 5+ sqrt[3]2\(5,0,0,2,1,0)&leftrightarrow (sqrt5+sqrt[3]2)^2\
    (2,5,15,0,0,3)&leftrightarrow (sqrt5+sqrt[3]2)^3
    end{align}$$



    And those four vectors are "obviously" linearly independent.






    share|cite|improve this answer











    $endgroup$



    Since you know that $[mathbb Q[sqrt[3]{2},sqrt 5]:mathbb Q] = 6$, you know that each of the six values $$1,sqrt{5},\sqrt[3]{2},sqrt[3]{2}sqrt{5},\sqrt[3]{4},sqrt[3]{4}sqrt 5tag{1}$$
    are linearly independent over $mathbb Q$.



    Now $$(sqrt[3]2+sqrt 5)^2=5cdot 1 + 2cdot sqrt[3]2sqrt 5 + 1cdotsqrt[3]4$$



    Is it possible for $1,sqrt[3]2+sqrt5,(sqrt[3]2+sqrt 5)^2$ to be linearly dependent over $mathbb Q$?



    Do the same with by adding the cube $(sqrt[3]2+sqrt5)^3$.



    Another way to look at it use (1) as a basis, and write elements of the field as:



    $$(a,b,c,d,e,f)to acdot 1 + bcdot sqrt5+csqrt[3]2+dsqrt[3]2sqrt5+esqrt[3]4+fsqrt[3]4sqrt5$$



    Then $$begin{align}(1,0,0,0,0,0)&leftrightarrow 1\(0,1,1,0,0,0)&leftrightarrow sqrt 5+ sqrt[3]2\(5,0,0,2,1,0)&leftrightarrow (sqrt5+sqrt[3]2)^2\
    (2,5,15,0,0,3)&leftrightarrow (sqrt5+sqrt[3]2)^3
    end{align}$$



    And those four vectors are "obviously" linearly independent.







    share|cite|improve this answer














    share|cite|improve this answer



    share|cite|improve this answer








    edited Jun 24 '15 at 2:16

























    answered Jun 24 '15 at 2:05









    Thomas AndrewsThomas Andrews

    130k12146298




    130k12146298








    • 1




      $begingroup$
      You don't need to go all the way up to $j=5$, since you know that it is either $1,2,3,$ or $6$. No reason to go beyond the cube. @orangeskid That conclusion was in the question. You only need to show that it is not a root of a quadratic, cubic or linear polynomial. That makes it very easy to prove linear independency of the above.
      $endgroup$
      – Thomas Andrews
      Jun 24 '15 at 2:33












    • $begingroup$
      Ah, OK, I get it. That's clever.
      $endgroup$
      – Orest Bucicovschi
      Jun 24 '15 at 2:43












    • $begingroup$
      I like this. I think I almost had it but didn't think to use the linear independence of the basis over $mathbb{Q}$.
      $endgroup$
      – TuoTuo
      Jun 24 '15 at 17:08
















    • 1




      $begingroup$
      You don't need to go all the way up to $j=5$, since you know that it is either $1,2,3,$ or $6$. No reason to go beyond the cube. @orangeskid That conclusion was in the question. You only need to show that it is not a root of a quadratic, cubic or linear polynomial. That makes it very easy to prove linear independency of the above.
      $endgroup$
      – Thomas Andrews
      Jun 24 '15 at 2:33












    • $begingroup$
      Ah, OK, I get it. That's clever.
      $endgroup$
      – Orest Bucicovschi
      Jun 24 '15 at 2:43












    • $begingroup$
      I like this. I think I almost had it but didn't think to use the linear independence of the basis over $mathbb{Q}$.
      $endgroup$
      – TuoTuo
      Jun 24 '15 at 17:08










    1




    1




    $begingroup$
    You don't need to go all the way up to $j=5$, since you know that it is either $1,2,3,$ or $6$. No reason to go beyond the cube. @orangeskid That conclusion was in the question. You only need to show that it is not a root of a quadratic, cubic or linear polynomial. That makes it very easy to prove linear independency of the above.
    $endgroup$
    – Thomas Andrews
    Jun 24 '15 at 2:33






    $begingroup$
    You don't need to go all the way up to $j=5$, since you know that it is either $1,2,3,$ or $6$. No reason to go beyond the cube. @orangeskid That conclusion was in the question. You only need to show that it is not a root of a quadratic, cubic or linear polynomial. That makes it very easy to prove linear independency of the above.
    $endgroup$
    – Thomas Andrews
    Jun 24 '15 at 2:33














    $begingroup$
    Ah, OK, I get it. That's clever.
    $endgroup$
    – Orest Bucicovschi
    Jun 24 '15 at 2:43






    $begingroup$
    Ah, OK, I get it. That's clever.
    $endgroup$
    – Orest Bucicovschi
    Jun 24 '15 at 2:43














    $begingroup$
    I like this. I think I almost had it but didn't think to use the linear independence of the basis over $mathbb{Q}$.
    $endgroup$
    – TuoTuo
    Jun 24 '15 at 17:08






    $begingroup$
    I like this. I think I almost had it but didn't think to use the linear independence of the basis over $mathbb{Q}$.
    $endgroup$
    – TuoTuo
    Jun 24 '15 at 17:08













    3












    $begingroup$

    Let's show that $s=sum sqrt[n_i]{d_i}$ ($d_i >0$ rationals ) generates $mathbb{Q}(sqrt[n_i]{d_i})$. Consider all in larger Galois extension $K supset mathbb{Q}(sqrt[n_i]{d_i})supset mathbb{Q}$. Now, to show that $s$ generates all the $sqrt[n_i]{d_i}$ it's enough to show that whenever a Galois transformation $phi$ of $K$ preserves $s$ it must preserve all the $sqrt[n_i]{d_i}$. Now any Galois transformation $phi$ takes $sqrt[n_i]{d_i}$ to some $omega_i sqrt[n_i]{d_i}$ where $omega_i^{n_i} =1$, so $phi(s) = sum omega_i sqrt[n_i]{d_i}$.
    Note that
    $$left|sum_i omega_i sqrt[n_i]{d_i}right| le sum_i sqrt[n_i]{d_i}$$ with equality
    if and only if $omega_i$ have the same argument. Therefore, if $sum_i omega_i sqrt[n_i]{d_i}= sum_i sqrt[n_i]{d_i}$ then all the $omega_i$ must be $1$.



    Conclusion: $phi(s) = s implies phi( sqrt[n_i]{d_i}) = sqrt[n_i]{d_i}$ for all $i$.



    $bf{Added:}$ A solution that uses mostly linear algebra, inspired from @
    Thomas Andrews: 's solution.



    We have the equalities
    begin{eqnarray}
    left(begin{array} {c}
    1\sqrt[3]{2}+sqrt{5}\ (sqrt[3]{2}+sqrt{5})^2\ (sqrt[3]{2}+sqrt{5})^3\ (sqrt[3]{2}+sqrt{5})^4\ (sqrt[3]{2}+sqrt{5})^5
    end{array}right) = left(
    begin{array}{cccccc}
    1 & 0 & 0 & 0 & 0 & 0 \
    0 & 1 & 0 & 1 & 0 & 0 \
    5 & 0 & 1 & 0 & 2 & 0 \
    2 & 15 & 0 & 5 & 0 & 3 \
    25 & 2 & 30 & 8 & 20 & 0 \
    100 & 125 & 2 & 25 & 10 & 50 \
    end{array}
    right) cdot left(begin{array} {c} 1 \ sqrt[3]{2}\ sqrt[3]{4} \ sqrt{5} \ sqrt[3]{2} sqrt{5} \sqrt[3]{4} sqrt{5}
    end{array}
    right)
    end{eqnarray}
    hence
    begin{eqnarray}
    left(begin{array} {c} 1 \ sqrt[3]{2}\ sqrt[3]{4} \ sqrt{5} \ sqrt[3]{2} sqrt{5} \sqrt[3]{4} sqrt{5}
    end{array}
    right)
    = left(
    begin{array}{cccccc}
    1 & 0 & 0 & 0 & 0 & 0 \
    frac{2275}{4054} & -frac{1714}{2027} & frac{195}{2027} & frac{500}{2027} & -frac{9}{4054} & -frac{30}{2027} \
    frac{2875}{2027} & -frac{1325}{2027} & -frac{955}{2027} & frac{150}{2027} & frac{100}{2027} & -frac{9}{2027} \
    -frac{2275}{4054} & frac{3741}{2027} & -frac{195}{2027} & -frac{500}{2027} & frac{9}{4054} & frac{30}{2027} \
    -frac{6505}{2027} & frac{1325}{4054} & frac{1491}{2027} & -frac{75}{2027} & -frac{50}{2027} & frac{9}{4054} \
    -frac{5143}{2027} & frac{2335}{2027} & -frac{650}{2027} & -frac{991}{2027} & frac{15}{2027} & frac{100}{2027} \
    end{array}
    right) cdot left(begin{array} {c}
    1\sqrt[3]{2}+sqrt{5}\ (sqrt[3]{2}+sqrt{5})^2\ (sqrt[3]{2}+sqrt{5})^3\ (sqrt[3]{2}+sqrt{5})^4\ (sqrt[3]{2}+sqrt{5})^5
    end{array}right)
    end{eqnarray}



    For instance, we have



    $
    tiny
    sqrt{5} = -frac{2275}{4054} + frac{3741}{2027} (sqrt[3]{2}+sqrt{5}) -frac{195}{2027}(sqrt[3]{2}+sqrt{5})^2 -frac{500}{2027} (sqrt[3]{2}+sqrt{5})^3+ frac{9}{4054} (sqrt[3]{2}+sqrt{5})^4+ frac{30}{2027} (sqrt[3]{2}+sqrt{5})^5$






    share|cite|improve this answer











    $endgroup$


















      3












      $begingroup$

      Let's show that $s=sum sqrt[n_i]{d_i}$ ($d_i >0$ rationals ) generates $mathbb{Q}(sqrt[n_i]{d_i})$. Consider all in larger Galois extension $K supset mathbb{Q}(sqrt[n_i]{d_i})supset mathbb{Q}$. Now, to show that $s$ generates all the $sqrt[n_i]{d_i}$ it's enough to show that whenever a Galois transformation $phi$ of $K$ preserves $s$ it must preserve all the $sqrt[n_i]{d_i}$. Now any Galois transformation $phi$ takes $sqrt[n_i]{d_i}$ to some $omega_i sqrt[n_i]{d_i}$ where $omega_i^{n_i} =1$, so $phi(s) = sum omega_i sqrt[n_i]{d_i}$.
      Note that
      $$left|sum_i omega_i sqrt[n_i]{d_i}right| le sum_i sqrt[n_i]{d_i}$$ with equality
      if and only if $omega_i$ have the same argument. Therefore, if $sum_i omega_i sqrt[n_i]{d_i}= sum_i sqrt[n_i]{d_i}$ then all the $omega_i$ must be $1$.



      Conclusion: $phi(s) = s implies phi( sqrt[n_i]{d_i}) = sqrt[n_i]{d_i}$ for all $i$.



      $bf{Added:}$ A solution that uses mostly linear algebra, inspired from @
      Thomas Andrews: 's solution.



      We have the equalities
      begin{eqnarray}
      left(begin{array} {c}
      1\sqrt[3]{2}+sqrt{5}\ (sqrt[3]{2}+sqrt{5})^2\ (sqrt[3]{2}+sqrt{5})^3\ (sqrt[3]{2}+sqrt{5})^4\ (sqrt[3]{2}+sqrt{5})^5
      end{array}right) = left(
      begin{array}{cccccc}
      1 & 0 & 0 & 0 & 0 & 0 \
      0 & 1 & 0 & 1 & 0 & 0 \
      5 & 0 & 1 & 0 & 2 & 0 \
      2 & 15 & 0 & 5 & 0 & 3 \
      25 & 2 & 30 & 8 & 20 & 0 \
      100 & 125 & 2 & 25 & 10 & 50 \
      end{array}
      right) cdot left(begin{array} {c} 1 \ sqrt[3]{2}\ sqrt[3]{4} \ sqrt{5} \ sqrt[3]{2} sqrt{5} \sqrt[3]{4} sqrt{5}
      end{array}
      right)
      end{eqnarray}
      hence
      begin{eqnarray}
      left(begin{array} {c} 1 \ sqrt[3]{2}\ sqrt[3]{4} \ sqrt{5} \ sqrt[3]{2} sqrt{5} \sqrt[3]{4} sqrt{5}
      end{array}
      right)
      = left(
      begin{array}{cccccc}
      1 & 0 & 0 & 0 & 0 & 0 \
      frac{2275}{4054} & -frac{1714}{2027} & frac{195}{2027} & frac{500}{2027} & -frac{9}{4054} & -frac{30}{2027} \
      frac{2875}{2027} & -frac{1325}{2027} & -frac{955}{2027} & frac{150}{2027} & frac{100}{2027} & -frac{9}{2027} \
      -frac{2275}{4054} & frac{3741}{2027} & -frac{195}{2027} & -frac{500}{2027} & frac{9}{4054} & frac{30}{2027} \
      -frac{6505}{2027} & frac{1325}{4054} & frac{1491}{2027} & -frac{75}{2027} & -frac{50}{2027} & frac{9}{4054} \
      -frac{5143}{2027} & frac{2335}{2027} & -frac{650}{2027} & -frac{991}{2027} & frac{15}{2027} & frac{100}{2027} \
      end{array}
      right) cdot left(begin{array} {c}
      1\sqrt[3]{2}+sqrt{5}\ (sqrt[3]{2}+sqrt{5})^2\ (sqrt[3]{2}+sqrt{5})^3\ (sqrt[3]{2}+sqrt{5})^4\ (sqrt[3]{2}+sqrt{5})^5
      end{array}right)
      end{eqnarray}



      For instance, we have



      $
      tiny
      sqrt{5} = -frac{2275}{4054} + frac{3741}{2027} (sqrt[3]{2}+sqrt{5}) -frac{195}{2027}(sqrt[3]{2}+sqrt{5})^2 -frac{500}{2027} (sqrt[3]{2}+sqrt{5})^3+ frac{9}{4054} (sqrt[3]{2}+sqrt{5})^4+ frac{30}{2027} (sqrt[3]{2}+sqrt{5})^5$






      share|cite|improve this answer











      $endgroup$
















        3












        3








        3





        $begingroup$

        Let's show that $s=sum sqrt[n_i]{d_i}$ ($d_i >0$ rationals ) generates $mathbb{Q}(sqrt[n_i]{d_i})$. Consider all in larger Galois extension $K supset mathbb{Q}(sqrt[n_i]{d_i})supset mathbb{Q}$. Now, to show that $s$ generates all the $sqrt[n_i]{d_i}$ it's enough to show that whenever a Galois transformation $phi$ of $K$ preserves $s$ it must preserve all the $sqrt[n_i]{d_i}$. Now any Galois transformation $phi$ takes $sqrt[n_i]{d_i}$ to some $omega_i sqrt[n_i]{d_i}$ where $omega_i^{n_i} =1$, so $phi(s) = sum omega_i sqrt[n_i]{d_i}$.
        Note that
        $$left|sum_i omega_i sqrt[n_i]{d_i}right| le sum_i sqrt[n_i]{d_i}$$ with equality
        if and only if $omega_i$ have the same argument. Therefore, if $sum_i omega_i sqrt[n_i]{d_i}= sum_i sqrt[n_i]{d_i}$ then all the $omega_i$ must be $1$.



        Conclusion: $phi(s) = s implies phi( sqrt[n_i]{d_i}) = sqrt[n_i]{d_i}$ for all $i$.



        $bf{Added:}$ A solution that uses mostly linear algebra, inspired from @
        Thomas Andrews: 's solution.



        We have the equalities
        begin{eqnarray}
        left(begin{array} {c}
        1\sqrt[3]{2}+sqrt{5}\ (sqrt[3]{2}+sqrt{5})^2\ (sqrt[3]{2}+sqrt{5})^3\ (sqrt[3]{2}+sqrt{5})^4\ (sqrt[3]{2}+sqrt{5})^5
        end{array}right) = left(
        begin{array}{cccccc}
        1 & 0 & 0 & 0 & 0 & 0 \
        0 & 1 & 0 & 1 & 0 & 0 \
        5 & 0 & 1 & 0 & 2 & 0 \
        2 & 15 & 0 & 5 & 0 & 3 \
        25 & 2 & 30 & 8 & 20 & 0 \
        100 & 125 & 2 & 25 & 10 & 50 \
        end{array}
        right) cdot left(begin{array} {c} 1 \ sqrt[3]{2}\ sqrt[3]{4} \ sqrt{5} \ sqrt[3]{2} sqrt{5} \sqrt[3]{4} sqrt{5}
        end{array}
        right)
        end{eqnarray}
        hence
        begin{eqnarray}
        left(begin{array} {c} 1 \ sqrt[3]{2}\ sqrt[3]{4} \ sqrt{5} \ sqrt[3]{2} sqrt{5} \sqrt[3]{4} sqrt{5}
        end{array}
        right)
        = left(
        begin{array}{cccccc}
        1 & 0 & 0 & 0 & 0 & 0 \
        frac{2275}{4054} & -frac{1714}{2027} & frac{195}{2027} & frac{500}{2027} & -frac{9}{4054} & -frac{30}{2027} \
        frac{2875}{2027} & -frac{1325}{2027} & -frac{955}{2027} & frac{150}{2027} & frac{100}{2027} & -frac{9}{2027} \
        -frac{2275}{4054} & frac{3741}{2027} & -frac{195}{2027} & -frac{500}{2027} & frac{9}{4054} & frac{30}{2027} \
        -frac{6505}{2027} & frac{1325}{4054} & frac{1491}{2027} & -frac{75}{2027} & -frac{50}{2027} & frac{9}{4054} \
        -frac{5143}{2027} & frac{2335}{2027} & -frac{650}{2027} & -frac{991}{2027} & frac{15}{2027} & frac{100}{2027} \
        end{array}
        right) cdot left(begin{array} {c}
        1\sqrt[3]{2}+sqrt{5}\ (sqrt[3]{2}+sqrt{5})^2\ (sqrt[3]{2}+sqrt{5})^3\ (sqrt[3]{2}+sqrt{5})^4\ (sqrt[3]{2}+sqrt{5})^5
        end{array}right)
        end{eqnarray}



        For instance, we have



        $
        tiny
        sqrt{5} = -frac{2275}{4054} + frac{3741}{2027} (sqrt[3]{2}+sqrt{5}) -frac{195}{2027}(sqrt[3]{2}+sqrt{5})^2 -frac{500}{2027} (sqrt[3]{2}+sqrt{5})^3+ frac{9}{4054} (sqrt[3]{2}+sqrt{5})^4+ frac{30}{2027} (sqrt[3]{2}+sqrt{5})^5$






        share|cite|improve this answer











        $endgroup$



        Let's show that $s=sum sqrt[n_i]{d_i}$ ($d_i >0$ rationals ) generates $mathbb{Q}(sqrt[n_i]{d_i})$. Consider all in larger Galois extension $K supset mathbb{Q}(sqrt[n_i]{d_i})supset mathbb{Q}$. Now, to show that $s$ generates all the $sqrt[n_i]{d_i}$ it's enough to show that whenever a Galois transformation $phi$ of $K$ preserves $s$ it must preserve all the $sqrt[n_i]{d_i}$. Now any Galois transformation $phi$ takes $sqrt[n_i]{d_i}$ to some $omega_i sqrt[n_i]{d_i}$ where $omega_i^{n_i} =1$, so $phi(s) = sum omega_i sqrt[n_i]{d_i}$.
        Note that
        $$left|sum_i omega_i sqrt[n_i]{d_i}right| le sum_i sqrt[n_i]{d_i}$$ with equality
        if and only if $omega_i$ have the same argument. Therefore, if $sum_i omega_i sqrt[n_i]{d_i}= sum_i sqrt[n_i]{d_i}$ then all the $omega_i$ must be $1$.



        Conclusion: $phi(s) = s implies phi( sqrt[n_i]{d_i}) = sqrt[n_i]{d_i}$ for all $i$.



        $bf{Added:}$ A solution that uses mostly linear algebra, inspired from @
        Thomas Andrews: 's solution.



        We have the equalities
        begin{eqnarray}
        left(begin{array} {c}
        1\sqrt[3]{2}+sqrt{5}\ (sqrt[3]{2}+sqrt{5})^2\ (sqrt[3]{2}+sqrt{5})^3\ (sqrt[3]{2}+sqrt{5})^4\ (sqrt[3]{2}+sqrt{5})^5
        end{array}right) = left(
        begin{array}{cccccc}
        1 & 0 & 0 & 0 & 0 & 0 \
        0 & 1 & 0 & 1 & 0 & 0 \
        5 & 0 & 1 & 0 & 2 & 0 \
        2 & 15 & 0 & 5 & 0 & 3 \
        25 & 2 & 30 & 8 & 20 & 0 \
        100 & 125 & 2 & 25 & 10 & 50 \
        end{array}
        right) cdot left(begin{array} {c} 1 \ sqrt[3]{2}\ sqrt[3]{4} \ sqrt{5} \ sqrt[3]{2} sqrt{5} \sqrt[3]{4} sqrt{5}
        end{array}
        right)
        end{eqnarray}
        hence
        begin{eqnarray}
        left(begin{array} {c} 1 \ sqrt[3]{2}\ sqrt[3]{4} \ sqrt{5} \ sqrt[3]{2} sqrt{5} \sqrt[3]{4} sqrt{5}
        end{array}
        right)
        = left(
        begin{array}{cccccc}
        1 & 0 & 0 & 0 & 0 & 0 \
        frac{2275}{4054} & -frac{1714}{2027} & frac{195}{2027} & frac{500}{2027} & -frac{9}{4054} & -frac{30}{2027} \
        frac{2875}{2027} & -frac{1325}{2027} & -frac{955}{2027} & frac{150}{2027} & frac{100}{2027} & -frac{9}{2027} \
        -frac{2275}{4054} & frac{3741}{2027} & -frac{195}{2027} & -frac{500}{2027} & frac{9}{4054} & frac{30}{2027} \
        -frac{6505}{2027} & frac{1325}{4054} & frac{1491}{2027} & -frac{75}{2027} & -frac{50}{2027} & frac{9}{4054} \
        -frac{5143}{2027} & frac{2335}{2027} & -frac{650}{2027} & -frac{991}{2027} & frac{15}{2027} & frac{100}{2027} \
        end{array}
        right) cdot left(begin{array} {c}
        1\sqrt[3]{2}+sqrt{5}\ (sqrt[3]{2}+sqrt{5})^2\ (sqrt[3]{2}+sqrt{5})^3\ (sqrt[3]{2}+sqrt{5})^4\ (sqrt[3]{2}+sqrt{5})^5
        end{array}right)
        end{eqnarray}



        For instance, we have



        $
        tiny
        sqrt{5} = -frac{2275}{4054} + frac{3741}{2027} (sqrt[3]{2}+sqrt{5}) -frac{195}{2027}(sqrt[3]{2}+sqrt{5})^2 -frac{500}{2027} (sqrt[3]{2}+sqrt{5})^3+ frac{9}{4054} (sqrt[3]{2}+sqrt{5})^4+ frac{30}{2027} (sqrt[3]{2}+sqrt{5})^5$







        share|cite|improve this answer














        share|cite|improve this answer



        share|cite|improve this answer








        edited Jun 24 '15 at 20:34

























        answered Jun 24 '15 at 2:41









        Orest BucicovschiOrest Bucicovschi

        28.5k31748




        28.5k31748























            3












            $begingroup$

            Brute-Force Method



            Let $alpha:=sqrt{5}+sqrt[3]{2}$. Then, $alpha^3-3sqrt{5}alpha^2+15alpha-5sqrt{5}=(alpha-sqrt{5})^3=2$, whence $$left(alpha^3+15alpha-2right)^3=5left(3alpha^2+5right)^2,,$$ or $alpha$ is a root of the polynomial $$f(x):=x^6-15x^4-4x^3+75x^2-60x-121 in mathbb{Q}[x],.$$ If this polynomial is reducible, then consider it modulo $3$, so $$f(x)=x^6+2x^3+2=left(x^2+2x+2right)^3$$ in $mathbb{F}_3$, and $x^2+2x+2$ is an irreducible element of $mathbb{F}_3[x]$. That is, $f(x)$ must have a monic quadratic factor $g(x)$ in $mathbb{Q}[x]$, and by Gauss's Lemma, $g(x)inmathbb{Z}[x]$. Hence, $g(x)=x^2+ax+b$ with $a,binmathbb{Z}$ and $bequiv 2 equiv -1pmod{3}$. Let $f(x)=g(x),h(x)$ for some $h(x)inmathbb{Q}[x]$ (which again yields $h(x)inmathbb{Z}[x]$). Since the coefficient of $x^5$ in $f(x)$ is $0$, $h(x)=x^4-ax^3+cx^2+dx+e$ for some $c,d,einmathbb{Z}$.



            Now, $$f(x)=x^6-4x^3+4=left(x^3-2right)^2=left(x^3+2^3right)^2=(x+2)^2left(x^2-2x+4right)^2$$ in $mathbb{F}_5$, where both $x+2$ and $x^2-2x+4$ are irreducible elements of $mathbb{F}_5[x]$. Hence, in $mathbb{F}_5$, either $$g(x)=(x+2)^2=x^2+4x+4text{ or }g(x)=x^2-2x+4,.$$ In either case, $bequiv 4equiv -1pmod{5}$. Together with $bequiv -1pmod{3}$, we deduce that $b equiv -1pmod{15}$. However, $b$ must divide the constant term $-121=-11^2$ of $f(x)$. This means $b=-1$ or $b=-121$.



            If $b=-1$, then $g(x)=x^2+ax-1$ and $h(x)=x^4-ax^3+cx^2+dx+121$. Equating the coefficients of $f(x)$ and $g(x),h(x)$, we have $$c-a^2-1=-15,,,, ac+a+d=-4,,$$ $$ad-c+121=75,,text{ and }121a-d=-60,.$$ Consequently, $c=a^2-14$ and $d=121a+60$, so that $$begin{align}0&=(ad-c+121)-75=ad-c+46\&=a(121a+60)-left(a^2-14right)+46=60left(2a^2+a+1right),.end{align}$$ However, $2a^2+a+1=0$ does not have an integer solution.



            If $b=-121$, $g(x)=x^2+ax-121$ and $h(x)=x^4-ax^3+cx^2+dx+1$. Equating the coefficients of $f(x)$ and $g(x),h(x)$, we have $$c-a^2-121=-15,,,, ac+121a+d=-4,,$$ $$ad-121c+1=75,,text{ and }a-121d=-60,.$$ Consequently, $c=a^2+106$ and $d=frac{a+60}{121}$, so that $$begin{align}0&=(ad-121c+1)-75=ad-121c-74\&=frac{a}{121}(a+60)-121left(a^2+106right)-75,,end{align}$$ or $$0=-frac{1}{121}left(14640a^2-60a+1561021right),.$$ However, $14640a^2-60a+1561021=0$ does not have an integer solution.



            We have derived a contradiction from the hypothesis that $f(x)$ is reducible over $mathbb{Q}$, so $f(x)$ must be irreducible in $mathbb{Q}[x]$. Hence, $mathbb{Q}(alpha)cong mathbb{Q}[x]/big(f(x)big)$ is a field extension of index $deg(f)=6$ over $mathbb{Q}$.






            share|cite|improve this answer











            $endgroup$













            • $begingroup$
              I think this is pretty complicated but appreciate the answer.
              $endgroup$
              – TuoTuo
              Jun 24 '15 at 17:00
















            3












            $begingroup$

            Brute-Force Method



            Let $alpha:=sqrt{5}+sqrt[3]{2}$. Then, $alpha^3-3sqrt{5}alpha^2+15alpha-5sqrt{5}=(alpha-sqrt{5})^3=2$, whence $$left(alpha^3+15alpha-2right)^3=5left(3alpha^2+5right)^2,,$$ or $alpha$ is a root of the polynomial $$f(x):=x^6-15x^4-4x^3+75x^2-60x-121 in mathbb{Q}[x],.$$ If this polynomial is reducible, then consider it modulo $3$, so $$f(x)=x^6+2x^3+2=left(x^2+2x+2right)^3$$ in $mathbb{F}_3$, and $x^2+2x+2$ is an irreducible element of $mathbb{F}_3[x]$. That is, $f(x)$ must have a monic quadratic factor $g(x)$ in $mathbb{Q}[x]$, and by Gauss's Lemma, $g(x)inmathbb{Z}[x]$. Hence, $g(x)=x^2+ax+b$ with $a,binmathbb{Z}$ and $bequiv 2 equiv -1pmod{3}$. Let $f(x)=g(x),h(x)$ for some $h(x)inmathbb{Q}[x]$ (which again yields $h(x)inmathbb{Z}[x]$). Since the coefficient of $x^5$ in $f(x)$ is $0$, $h(x)=x^4-ax^3+cx^2+dx+e$ for some $c,d,einmathbb{Z}$.



            Now, $$f(x)=x^6-4x^3+4=left(x^3-2right)^2=left(x^3+2^3right)^2=(x+2)^2left(x^2-2x+4right)^2$$ in $mathbb{F}_5$, where both $x+2$ and $x^2-2x+4$ are irreducible elements of $mathbb{F}_5[x]$. Hence, in $mathbb{F}_5$, either $$g(x)=(x+2)^2=x^2+4x+4text{ or }g(x)=x^2-2x+4,.$$ In either case, $bequiv 4equiv -1pmod{5}$. Together with $bequiv -1pmod{3}$, we deduce that $b equiv -1pmod{15}$. However, $b$ must divide the constant term $-121=-11^2$ of $f(x)$. This means $b=-1$ or $b=-121$.



            If $b=-1$, then $g(x)=x^2+ax-1$ and $h(x)=x^4-ax^3+cx^2+dx+121$. Equating the coefficients of $f(x)$ and $g(x),h(x)$, we have $$c-a^2-1=-15,,,, ac+a+d=-4,,$$ $$ad-c+121=75,,text{ and }121a-d=-60,.$$ Consequently, $c=a^2-14$ and $d=121a+60$, so that $$begin{align}0&=(ad-c+121)-75=ad-c+46\&=a(121a+60)-left(a^2-14right)+46=60left(2a^2+a+1right),.end{align}$$ However, $2a^2+a+1=0$ does not have an integer solution.



            If $b=-121$, $g(x)=x^2+ax-121$ and $h(x)=x^4-ax^3+cx^2+dx+1$. Equating the coefficients of $f(x)$ and $g(x),h(x)$, we have $$c-a^2-121=-15,,,, ac+121a+d=-4,,$$ $$ad-121c+1=75,,text{ and }a-121d=-60,.$$ Consequently, $c=a^2+106$ and $d=frac{a+60}{121}$, so that $$begin{align}0&=(ad-121c+1)-75=ad-121c-74\&=frac{a}{121}(a+60)-121left(a^2+106right)-75,,end{align}$$ or $$0=-frac{1}{121}left(14640a^2-60a+1561021right),.$$ However, $14640a^2-60a+1561021=0$ does not have an integer solution.



            We have derived a contradiction from the hypothesis that $f(x)$ is reducible over $mathbb{Q}$, so $f(x)$ must be irreducible in $mathbb{Q}[x]$. Hence, $mathbb{Q}(alpha)cong mathbb{Q}[x]/big(f(x)big)$ is a field extension of index $deg(f)=6$ over $mathbb{Q}$.






            share|cite|improve this answer











            $endgroup$













            • $begingroup$
              I think this is pretty complicated but appreciate the answer.
              $endgroup$
              – TuoTuo
              Jun 24 '15 at 17:00














            3












            3








            3





            $begingroup$

            Brute-Force Method



            Let $alpha:=sqrt{5}+sqrt[3]{2}$. Then, $alpha^3-3sqrt{5}alpha^2+15alpha-5sqrt{5}=(alpha-sqrt{5})^3=2$, whence $$left(alpha^3+15alpha-2right)^3=5left(3alpha^2+5right)^2,,$$ or $alpha$ is a root of the polynomial $$f(x):=x^6-15x^4-4x^3+75x^2-60x-121 in mathbb{Q}[x],.$$ If this polynomial is reducible, then consider it modulo $3$, so $$f(x)=x^6+2x^3+2=left(x^2+2x+2right)^3$$ in $mathbb{F}_3$, and $x^2+2x+2$ is an irreducible element of $mathbb{F}_3[x]$. That is, $f(x)$ must have a monic quadratic factor $g(x)$ in $mathbb{Q}[x]$, and by Gauss's Lemma, $g(x)inmathbb{Z}[x]$. Hence, $g(x)=x^2+ax+b$ with $a,binmathbb{Z}$ and $bequiv 2 equiv -1pmod{3}$. Let $f(x)=g(x),h(x)$ for some $h(x)inmathbb{Q}[x]$ (which again yields $h(x)inmathbb{Z}[x]$). Since the coefficient of $x^5$ in $f(x)$ is $0$, $h(x)=x^4-ax^3+cx^2+dx+e$ for some $c,d,einmathbb{Z}$.



            Now, $$f(x)=x^6-4x^3+4=left(x^3-2right)^2=left(x^3+2^3right)^2=(x+2)^2left(x^2-2x+4right)^2$$ in $mathbb{F}_5$, where both $x+2$ and $x^2-2x+4$ are irreducible elements of $mathbb{F}_5[x]$. Hence, in $mathbb{F}_5$, either $$g(x)=(x+2)^2=x^2+4x+4text{ or }g(x)=x^2-2x+4,.$$ In either case, $bequiv 4equiv -1pmod{5}$. Together with $bequiv -1pmod{3}$, we deduce that $b equiv -1pmod{15}$. However, $b$ must divide the constant term $-121=-11^2$ of $f(x)$. This means $b=-1$ or $b=-121$.



            If $b=-1$, then $g(x)=x^2+ax-1$ and $h(x)=x^4-ax^3+cx^2+dx+121$. Equating the coefficients of $f(x)$ and $g(x),h(x)$, we have $$c-a^2-1=-15,,,, ac+a+d=-4,,$$ $$ad-c+121=75,,text{ and }121a-d=-60,.$$ Consequently, $c=a^2-14$ and $d=121a+60$, so that $$begin{align}0&=(ad-c+121)-75=ad-c+46\&=a(121a+60)-left(a^2-14right)+46=60left(2a^2+a+1right),.end{align}$$ However, $2a^2+a+1=0$ does not have an integer solution.



            If $b=-121$, $g(x)=x^2+ax-121$ and $h(x)=x^4-ax^3+cx^2+dx+1$. Equating the coefficients of $f(x)$ and $g(x),h(x)$, we have $$c-a^2-121=-15,,,, ac+121a+d=-4,,$$ $$ad-121c+1=75,,text{ and }a-121d=-60,.$$ Consequently, $c=a^2+106$ and $d=frac{a+60}{121}$, so that $$begin{align}0&=(ad-121c+1)-75=ad-121c-74\&=frac{a}{121}(a+60)-121left(a^2+106right)-75,,end{align}$$ or $$0=-frac{1}{121}left(14640a^2-60a+1561021right),.$$ However, $14640a^2-60a+1561021=0$ does not have an integer solution.



            We have derived a contradiction from the hypothesis that $f(x)$ is reducible over $mathbb{Q}$, so $f(x)$ must be irreducible in $mathbb{Q}[x]$. Hence, $mathbb{Q}(alpha)cong mathbb{Q}[x]/big(f(x)big)$ is a field extension of index $deg(f)=6$ over $mathbb{Q}$.






            share|cite|improve this answer











            $endgroup$



            Brute-Force Method



            Let $alpha:=sqrt{5}+sqrt[3]{2}$. Then, $alpha^3-3sqrt{5}alpha^2+15alpha-5sqrt{5}=(alpha-sqrt{5})^3=2$, whence $$left(alpha^3+15alpha-2right)^3=5left(3alpha^2+5right)^2,,$$ or $alpha$ is a root of the polynomial $$f(x):=x^6-15x^4-4x^3+75x^2-60x-121 in mathbb{Q}[x],.$$ If this polynomial is reducible, then consider it modulo $3$, so $$f(x)=x^6+2x^3+2=left(x^2+2x+2right)^3$$ in $mathbb{F}_3$, and $x^2+2x+2$ is an irreducible element of $mathbb{F}_3[x]$. That is, $f(x)$ must have a monic quadratic factor $g(x)$ in $mathbb{Q}[x]$, and by Gauss's Lemma, $g(x)inmathbb{Z}[x]$. Hence, $g(x)=x^2+ax+b$ with $a,binmathbb{Z}$ and $bequiv 2 equiv -1pmod{3}$. Let $f(x)=g(x),h(x)$ for some $h(x)inmathbb{Q}[x]$ (which again yields $h(x)inmathbb{Z}[x]$). Since the coefficient of $x^5$ in $f(x)$ is $0$, $h(x)=x^4-ax^3+cx^2+dx+e$ for some $c,d,einmathbb{Z}$.



            Now, $$f(x)=x^6-4x^3+4=left(x^3-2right)^2=left(x^3+2^3right)^2=(x+2)^2left(x^2-2x+4right)^2$$ in $mathbb{F}_5$, where both $x+2$ and $x^2-2x+4$ are irreducible elements of $mathbb{F}_5[x]$. Hence, in $mathbb{F}_5$, either $$g(x)=(x+2)^2=x^2+4x+4text{ or }g(x)=x^2-2x+4,.$$ In either case, $bequiv 4equiv -1pmod{5}$. Together with $bequiv -1pmod{3}$, we deduce that $b equiv -1pmod{15}$. However, $b$ must divide the constant term $-121=-11^2$ of $f(x)$. This means $b=-1$ or $b=-121$.



            If $b=-1$, then $g(x)=x^2+ax-1$ and $h(x)=x^4-ax^3+cx^2+dx+121$. Equating the coefficients of $f(x)$ and $g(x),h(x)$, we have $$c-a^2-1=-15,,,, ac+a+d=-4,,$$ $$ad-c+121=75,,text{ and }121a-d=-60,.$$ Consequently, $c=a^2-14$ and $d=121a+60$, so that $$begin{align}0&=(ad-c+121)-75=ad-c+46\&=a(121a+60)-left(a^2-14right)+46=60left(2a^2+a+1right),.end{align}$$ However, $2a^2+a+1=0$ does not have an integer solution.



            If $b=-121$, $g(x)=x^2+ax-121$ and $h(x)=x^4-ax^3+cx^2+dx+1$. Equating the coefficients of $f(x)$ and $g(x),h(x)$, we have $$c-a^2-121=-15,,,, ac+121a+d=-4,,$$ $$ad-121c+1=75,,text{ and }a-121d=-60,.$$ Consequently, $c=a^2+106$ and $d=frac{a+60}{121}$, so that $$begin{align}0&=(ad-121c+1)-75=ad-121c-74\&=frac{a}{121}(a+60)-121left(a^2+106right)-75,,end{align}$$ or $$0=-frac{1}{121}left(14640a^2-60a+1561021right),.$$ However, $14640a^2-60a+1561021=0$ does not have an integer solution.



            We have derived a contradiction from the hypothesis that $f(x)$ is reducible over $mathbb{Q}$, so $f(x)$ must be irreducible in $mathbb{Q}[x]$. Hence, $mathbb{Q}(alpha)cong mathbb{Q}[x]/big(f(x)big)$ is a field extension of index $deg(f)=6$ over $mathbb{Q}$.







            share|cite|improve this answer














            share|cite|improve this answer



            share|cite|improve this answer








            edited Dec 18 '18 at 0:39

























            answered Jun 24 '15 at 3:01









            BatominovskiBatominovski

            33.1k33293




            33.1k33293












            • $begingroup$
              I think this is pretty complicated but appreciate the answer.
              $endgroup$
              – TuoTuo
              Jun 24 '15 at 17:00


















            • $begingroup$
              I think this is pretty complicated but appreciate the answer.
              $endgroup$
              – TuoTuo
              Jun 24 '15 at 17:00
















            $begingroup$
            I think this is pretty complicated but appreciate the answer.
            $endgroup$
            – TuoTuo
            Jun 24 '15 at 17:00




            $begingroup$
            I think this is pretty complicated but appreciate the answer.
            $endgroup$
            – TuoTuo
            Jun 24 '15 at 17:00











            2












            $begingroup$

            Clearly $LHSsubseteq RHS$. Now it suffices to write down the minimal polynomial of $sqrt{5}+sqrt[3]{2}$ and note it has degree $6$.






            share|cite|improve this answer









            $endgroup$













            • $begingroup$
              This would be my method, too. You get a polynomial of degree $6$, but you must also show that it’s irreducible.
              $endgroup$
              – Lubin
              Jun 24 '15 at 16:40










            • $begingroup$
              Claiming that the minimal polynomial has degree 6 with no justification is almost like assuming that these extensions are equal.
              $endgroup$
              – TuoTuo
              Jun 24 '15 at 16:47










            • $begingroup$
              Isn't there a general result saying that if $a$ has deg 2 min poly and $b$ has deg $3$ min poly, then $a+b$ has deg 6 min poly?
              $endgroup$
              – Damian Reding
              Jun 24 '15 at 17:33










            • $begingroup$
              @Mathgemnini There very well may be but I've never seen it and a quick google search didn't reveal it. In either case I doubt we could get away with using that theorem on the qual (if such a theorem exists) since it basically trivializes the problem and is not a major result.
              $endgroup$
              – TuoTuo
              Jun 24 '15 at 20:52
















            2












            $begingroup$

            Clearly $LHSsubseteq RHS$. Now it suffices to write down the minimal polynomial of $sqrt{5}+sqrt[3]{2}$ and note it has degree $6$.






            share|cite|improve this answer









            $endgroup$













            • $begingroup$
              This would be my method, too. You get a polynomial of degree $6$, but you must also show that it’s irreducible.
              $endgroup$
              – Lubin
              Jun 24 '15 at 16:40










            • $begingroup$
              Claiming that the minimal polynomial has degree 6 with no justification is almost like assuming that these extensions are equal.
              $endgroup$
              – TuoTuo
              Jun 24 '15 at 16:47










            • $begingroup$
              Isn't there a general result saying that if $a$ has deg 2 min poly and $b$ has deg $3$ min poly, then $a+b$ has deg 6 min poly?
              $endgroup$
              – Damian Reding
              Jun 24 '15 at 17:33










            • $begingroup$
              @Mathgemnini There very well may be but I've never seen it and a quick google search didn't reveal it. In either case I doubt we could get away with using that theorem on the qual (if such a theorem exists) since it basically trivializes the problem and is not a major result.
              $endgroup$
              – TuoTuo
              Jun 24 '15 at 20:52














            2












            2








            2





            $begingroup$

            Clearly $LHSsubseteq RHS$. Now it suffices to write down the minimal polynomial of $sqrt{5}+sqrt[3]{2}$ and note it has degree $6$.






            share|cite|improve this answer









            $endgroup$



            Clearly $LHSsubseteq RHS$. Now it suffices to write down the minimal polynomial of $sqrt{5}+sqrt[3]{2}$ and note it has degree $6$.







            share|cite|improve this answer












            share|cite|improve this answer



            share|cite|improve this answer










            answered Jun 24 '15 at 2:00









            Damian RedingDamian Reding

            5,21911834




            5,21911834












            • $begingroup$
              This would be my method, too. You get a polynomial of degree $6$, but you must also show that it’s irreducible.
              $endgroup$
              – Lubin
              Jun 24 '15 at 16:40










            • $begingroup$
              Claiming that the minimal polynomial has degree 6 with no justification is almost like assuming that these extensions are equal.
              $endgroup$
              – TuoTuo
              Jun 24 '15 at 16:47










            • $begingroup$
              Isn't there a general result saying that if $a$ has deg 2 min poly and $b$ has deg $3$ min poly, then $a+b$ has deg 6 min poly?
              $endgroup$
              – Damian Reding
              Jun 24 '15 at 17:33










            • $begingroup$
              @Mathgemnini There very well may be but I've never seen it and a quick google search didn't reveal it. In either case I doubt we could get away with using that theorem on the qual (if such a theorem exists) since it basically trivializes the problem and is not a major result.
              $endgroup$
              – TuoTuo
              Jun 24 '15 at 20:52


















            • $begingroup$
              This would be my method, too. You get a polynomial of degree $6$, but you must also show that it’s irreducible.
              $endgroup$
              – Lubin
              Jun 24 '15 at 16:40










            • $begingroup$
              Claiming that the minimal polynomial has degree 6 with no justification is almost like assuming that these extensions are equal.
              $endgroup$
              – TuoTuo
              Jun 24 '15 at 16:47










            • $begingroup$
              Isn't there a general result saying that if $a$ has deg 2 min poly and $b$ has deg $3$ min poly, then $a+b$ has deg 6 min poly?
              $endgroup$
              – Damian Reding
              Jun 24 '15 at 17:33










            • $begingroup$
              @Mathgemnini There very well may be but I've never seen it and a quick google search didn't reveal it. In either case I doubt we could get away with using that theorem on the qual (if such a theorem exists) since it basically trivializes the problem and is not a major result.
              $endgroup$
              – TuoTuo
              Jun 24 '15 at 20:52
















            $begingroup$
            This would be my method, too. You get a polynomial of degree $6$, but you must also show that it’s irreducible.
            $endgroup$
            – Lubin
            Jun 24 '15 at 16:40




            $begingroup$
            This would be my method, too. You get a polynomial of degree $6$, but you must also show that it’s irreducible.
            $endgroup$
            – Lubin
            Jun 24 '15 at 16:40












            $begingroup$
            Claiming that the minimal polynomial has degree 6 with no justification is almost like assuming that these extensions are equal.
            $endgroup$
            – TuoTuo
            Jun 24 '15 at 16:47




            $begingroup$
            Claiming that the minimal polynomial has degree 6 with no justification is almost like assuming that these extensions are equal.
            $endgroup$
            – TuoTuo
            Jun 24 '15 at 16:47












            $begingroup$
            Isn't there a general result saying that if $a$ has deg 2 min poly and $b$ has deg $3$ min poly, then $a+b$ has deg 6 min poly?
            $endgroup$
            – Damian Reding
            Jun 24 '15 at 17:33




            $begingroup$
            Isn't there a general result saying that if $a$ has deg 2 min poly and $b$ has deg $3$ min poly, then $a+b$ has deg 6 min poly?
            $endgroup$
            – Damian Reding
            Jun 24 '15 at 17:33












            $begingroup$
            @Mathgemnini There very well may be but I've never seen it and a quick google search didn't reveal it. In either case I doubt we could get away with using that theorem on the qual (if such a theorem exists) since it basically trivializes the problem and is not a major result.
            $endgroup$
            – TuoTuo
            Jun 24 '15 at 20:52




            $begingroup$
            @Mathgemnini There very well may be but I've never seen it and a quick google search didn't reveal it. In either case I doubt we could get away with using that theorem on the qual (if such a theorem exists) since it basically trivializes the problem and is not a major result.
            $endgroup$
            – TuoTuo
            Jun 24 '15 at 20:52


















            draft saved

            draft discarded




















































            Thanks for contributing an answer to Mathematics Stack Exchange!


            • Please be sure to answer the question. Provide details and share your research!

            But avoid



            • Asking for help, clarification, or responding to other answers.

            • Making statements based on opinion; back them up with references or personal experience.


            Use MathJax to format equations. MathJax reference.


            To learn more, see our tips on writing great answers.




            draft saved


            draft discarded














            StackExchange.ready(
            function () {
            StackExchange.openid.initPostLogin('.new-post-login', 'https%3a%2f%2fmath.stackexchange.com%2fquestions%2f1337071%2fshow-that-mathbbq-sqrt5-sqrt32-mathbbq-sqrt5-sqrt32%23new-answer', 'question_page');
            }
            );

            Post as a guest















            Required, but never shown





















































            Required, but never shown














            Required, but never shown












            Required, but never shown







            Required, but never shown

































            Required, but never shown














            Required, but never shown












            Required, but never shown







            Required, but never shown







            Popular posts from this blog

            How do I know what Microsoft account the skydrive app is syncing to?

            When does type information flow backwards in C++?

            Grease: Live!